Solving a 2D PDE using the Fourier Transform

Click For Summary
The discussion focuses on solving a 2D partial differential equation (PDE) using the Fourier Transform, with an emphasis on the initial condition. The initial attempt involved transforming both spatial and temporal variables, leading to a simplification that resulted in the equation ω = k. However, it was noted that this approach does not adequately incorporate the initial condition, suggesting that the Fourier Transform is not suitable for time variables constrained to t ≥ 0. Instead, the Laplace Transform was recommended as a more appropriate method, allowing for the acknowledgment of initial conditions. The conversation concluded with clarification that while the Fourier Transform can be applied, it may require additional considerations for time-dependent terms.
CGandC
Messages
326
Reaction score
34

Homework Statement


Solve the following partial differential equation , using Fourier Transform:
upload_2018-11-28_20-50-28.png


Given the following:
upload_2018-11-28_20-50-47.png


And a initial condition:
upload_2018-11-28_20-51-5.png

Homework Equations

The Attempt at a Solution



First , i associate spectral variables to the x and t variables:
## k ## is the spectral variable corresponding to ## x ##
## \omega ## is the spectral variable corresponding to ## t ##

Using Fourier transform on the PDE , i get:

## (i\omega)(ik)\widetilde{u(\omega,k)}=(ik)^2 \widetilde{u(\omega,k)} ##

After simplifying , I get : ## \omega = k ##

How am I supposed to proceed from here? ( I didn't find ## \widetilde{u(\omega,k)} ## )
 

Attachments

  • upload_2018-11-28_20-50-28.png
    upload_2018-11-28_20-50-28.png
    5.5 KB · Views: 1,007
  • upload_2018-11-28_20-50-47.png
    upload_2018-11-28_20-50-47.png
    3.3 KB · Views: 915
  • upload_2018-11-28_20-51-5.png
    upload_2018-11-28_20-51-5.png
    5 KB · Views: 886
Physics news on Phys.org
CGandC said:

Homework Statement


Solve the following partial differential equation , using Fourier Transform:
View attachment 234811

Given the following:
View attachment 234812

And a initial condition:
View attachment 234813

Homework Equations

The Attempt at a Solution



First , i associate spectral variables to the x and t variables:
## k ## is the spectral variable corresponding to ## x ##
## \omega ## is the spectral variable corresponding to ## t ##

Using Fourier transform on the PDE , i get:

## (i\omega)(ik)\widetilde{u(\omega,k)}=(ik)^2 \widetilde{u(\omega,k)} ##

After simplifying , I get : ## \omega = k ##

How am I supposed to proceed from here? ( I didn't find ## \widetilde{u(\omega,k)} ## )

Your method fails because it has no straightforward way to incorporate the initial condition on ##u(x,0).## If I were doing the problem I would just take the ##x##-transform, and write
$$u(x,t) = \int_R \tilde{u}(k,t) e^{-ik x} \, dk$$ and so transform the PDE to
$$(-ik) \frac{\partial \tilde{u}(k,t)}{\partial t} = (-ik)^2 \tilde{u}(k,t), $$
with ##\tilde{u}(k,0) = ## Fourier transform of the given function ##u(x,0).##

However, if you really insist on transforming both variables ##x## and ##t## you should recognize that the Fourier transform does not fit well with the restriction ##t \geq 0.## It would be better to use the Laplace transform
$$g(s) =( {\cal L} f)(s) = \int_0^\infty e^{-st} f(t) \, dt,$$
and to recognize the possibility of acknowledging the initial condition through the standard property
$$({\cal L}\, dt/dt)(s) = s ({\cal L} f)(s) - f(0).$$ You will get a solvable transformed system: try it and see.
 
  • Like
Likes CGandC and Delta2
Ray Vickson said:
Your method fails because it has no straightforward way to incorporate the initial condition on ##u(x,0).## If I were doing the problem I would just take the ##x##-transform, and write
$$u(x,t) = \int_R \tilde{u}(k,t) e^{-ik x} \, dk$$ and so transform the PDE to
$$(-ik) \frac{\partial \tilde{u}(k,t)}{\partial t} = (-ik)^2 \tilde{u}(k,t), $$
with ##\tilde{u}(k,0) = ## Fourier transform of the given function ##u(x,0).##

However, if you really insist on transforming both variables ##x## and ##t## you should recognize that the Fourier transform does not fit well with the restriction ##t \geq 0.## It would be better to use the Laplace transform
$$g(s) =( {\cal L} f)(s) = \int_0^\infty e^{-st} f(t) \, dt,$$
and to recognize the possibility of acknowledging the initial condition through the standard property
$$({\cal L}\, dt/dt)(s) = s ({\cal L} f)(s) - f(0).$$ You will get a solvable transformed system: try it and see.

I managed to solve the question , like you meant - using the restriction that the time variable ## t ## is not Fourier transformable ( meaning: I only use Fourier transform on the x variable ) .

But I don't fully understand why the time variable ## t ## is not Fourier transformable , can you please elaborate?
 
CGandC said:
I managed to solve the question , like you meant - using the restriction that the time variable ## t ## is not Fourier transformable ( meaning: I only use Fourier transform on the x variable ) .

But I don't fully understand why the time variable ## t ## is not Fourier transformable , can you please elaborate?
I did not say it was not Fourier transformable; I said that the Fourier transform does not fit well in that case.

Of course, you can always take the F.T. of a function of the form
$$F(t) = \begin{cases} 0 & t < 0\\
f(t) & t \geq 0
\end{cases} $$
because you can perform the Fourier integral. However, such problems fit more nicely into the area of Laplace transforms (which are, essentially, Fourier transforms at a complex argument ##k##). But go ahead and do the Fourier transform if you want to; just remember that the formula for the transform of ##df/dt## will not be what you think it is. You will need extra terms corresponding to the value of ##f(0)##. I will let you work it out for yourself, or look it up somewhere.
 
Question: A clock's minute hand has length 4 and its hour hand has length 3. What is the distance between the tips at the moment when it is increasing most rapidly?(Putnam Exam Question) Answer: Making assumption that both the hands moves at constant angular velocities, the answer is ## \sqrt{7} .## But don't you think this assumption is somewhat doubtful and wrong?

Similar threads

Replies
1
Views
2K
  • · Replies 1 ·
Replies
1
Views
2K
  • · Replies 5 ·
Replies
5
Views
2K
  • · Replies 3 ·
Replies
3
Views
2K
  • · Replies 11 ·
Replies
11
Views
2K
Replies
5
Views
2K
  • · Replies 12 ·
Replies
12
Views
2K
  • · Replies 9 ·
Replies
9
Views
2K
  • · Replies 3 ·
Replies
3
Views
3K
Replies
2
Views
2K